LSAT and Law School Admissions Forum

Get expert LSAT preparation and law school admissions advice from PowerScore Test Preparation.

 lenihil
  • Posts: 35
  • Joined: Apr 27, 2020
|
#77679
Dear PowerScore,

Could you please explain why answer choice (B) is wrong?

Is that because Anika didn't say whether Inez should adopt the proposed plan or not, but instead, Anika just disagreed with Inez's prediction? Or, is that because Anika didn't think Inez's plan will be effective?

Thank you for your help.
 Luke Haqq
PowerScore Staff
  • PowerScore Staff
  • Posts: 722
  • Joined: Apr 26, 2012
|
#77783
Hi lenihil!

To understand why answer choice (B) is incorrect and (A) correct for this one, let's take a look at both answer choices.

First, let's start with answer (A): "indicating that a particular plan would have an effect contrary to the anticipated effect." The question stem tells us that this is a method of reasoning type of question. In order to feel confident that you've chosen the correct answer on questions like this, it's important to take things that are stated generally or generically (e.g., "a particular plan") and try to give that statement substance based on the information in the passage (which specific plan is being discussed?).

We could bracket the specific information from the stimulus that corresponds to these generic words: Anika proceeds by "indicating that a particular plan [Inez's plan to get appraisers] would have an effect contrary [people would be less willing to buy because things are needlessly more costly] to the anticipated effect [the effect predicted by Inez of people being more willing to buy]." Answer choice (A) fits exactly with what Anika's response is doing--Anika claims that the plan to get appraisers would not make customers more willing to buy, because this would just drive up the price needlessly when customers are already experts enough at appraising on their own.

By contrast, answer choice (B) states that Anika proceeds by "claiming that a particular plan should not be adopted because, while effective, it would have at least one undesirable consequence." Regarding answer (B), you write:
Is that because Anika didn't say whether Inez should adopt the proposed plan or not, but instead, Anika just disagreed with Inez's prediction? Or, is that because Anika didn't think Inez's plan will be effective?
I think that both of your intuitions are correct. That is, the stimulus doesn't actually say anywhere that Anika thinks that Inez's plan will be effective. That is an example of something that should be double-checked against the stimulus. Answer choice (B) would probably be a good answer if it only eliminated the words "while effective"; since Anika doesn't explicitly acknowledge that Inez's plan would be effective, however, those two words are enough to make the answer choice incorrect because it doesn't accurately describe Anika's method of argumentation.
 lenihil
  • Posts: 35
  • Joined: Apr 27, 2020
|
#77801
Dear Luke,

Thank you very much. It is all clear now.

Harder LSAT questions, especially the ones contain exchanges and necessary assumptions, are always tough and subtle! :cry: :cry: I will work harder. :ras: :ras: Thank you.
 flowskiferda
  • Posts: 30
  • Joined: Sep 19, 2020
|
#83896
How can A be correct? Never does she indicate that it would have a contrary effect; she simply says it would have an additional unintended effect (raising prices). I've looked up the definition of contrary in 5 different dictionaries, and they all agree that contrary means "opposite". So in this case, since the effect that Inez predicts would be more customers, the contrary effect would be fewer customers. Unless we are to make the assumption, external from the stimulus, that increased prices will surely mean fewer customers (a pretty questionable assumption considering we know nothing about the elasticity of the demand for antiques), there's no way that A can be right.
 Adam Tyson
PowerScore Staff
  • PowerScore Staff
  • Posts: 5153
  • Joined: Apr 14, 2011
|
#83923
And yet it is the correct answer, flowskiferda! So your goal should be to figure out why LSAC thought this was the best response, rather than focusing on finding fault with it. Keep in mind two things while doing this: 1) the "best" answer is a relative thing - the credited response only needs to be better than the other four answers, and doesn't have to be perfect; 2) the argument began with a reference to "these poor economic times," which suggests pretty strongly that higher prices would have a deleterious effect. Maybe not complete proof, but again, we don't need a perfect answer, just the best one of the five presented.

Your concern is valid, and there are some assumptions built into that answer, but those assumptions aren't entirely unreasonable and it does seem that Anika opposed the plan not simply because it won't help but because it would actually hurt. If Anika did not believe that the plan would cause a problem, why is she even objecting? What's the point of her raising the issue of increased prices if not that she is concerned it will drive sales down?

A recent blog post might be of interest to you, and to others encountering similar issues with correct answers that are in some way objectionable. Check this out:

https://blog.powerscore.com/lsat/you-ca ... -the-lsat/

Check that out, and be prepared on this test to sometimes accept that the right answer might not be everything we want it to be. It can be a bitter pill, but it's good medicine for what ails us!

Get the most out of your LSAT Prep Plus subscription.

Analyze and track your performance with our Testing and Analytics Package.